the fraction 1-4 is blank to the decimal of 0.25

Answers

Answer 1

Answer:

equal

Step-by-step explanation:

1/4 = .25

1÷4=.25

Answer 2
equal/equivalent hope this helps:)

Related Questions

During a snowstorm, Grayson tracked the amount of snow on the ground. When the storm began, there were 4 inches of snow on the ground. For the first 3 hours of the storm, snow fell at a constant rate of 1 inch per hour. The storm then stopped for 5 hours and then started again at a constant rate of 3 inches per hour for the next 2 hours. As soon as the storm stopped again, the sun came out and melted the snow for the next 2 hours at a constant rate of 4 inches per hour. Make a graph showing the inches of snow on the ground over time using the data that Grayson collected.

Answers

We can plot all that happened in the next graph:

This is the graph showing the inches of the snow on the ground over time using the data that Grayson collected.

Use the graphing tool to determine the true statementsregarding the represented function. Check all that apply.f(x) > 0 over the interval (1,).Of(x) < 0 over the interval [1,0).Of(x) 0 over the interval (-∞, 1].Of(x) > 0 over the interval (-∞, 1).Of(x) > 0 over the interval (-∞o).Intro2010-202

Answers

The true statements are,

f(x) > 0 over the interval (1, ∞)

f(x) ≤ 0 over the interval (-∞, 1]

Interval of a function:

If the value of the function f (x) rises as the value of x rises, the function interval is said to be positive. Instead, if the value of the function f (x) drops as the value of x increases, the function interval is said to be negative.

If the endpoints are absent from an interval, it is referred to as being open. It's indicated by ( ). Examples are (1, 2), which denotes larger than 1 and less than 2. Any interval that contains all the limit points is said to be closed. The symbol for it is []. For instance, [2, 5] denotes a value greater or equal to 2 and lower or equal to 5. If one of an open interval's endpoints is present, it is referred to as a half-open interval.

To learn more about interval of a function visit:

brainly.com/question/12223134

#SPJ9

Given y=0.5x^2, describe the transformation (x,y) --> (x,4y) and sketch the graph of this image

Answers

We are given the equation y = 0.5x^2. To describe its transformation from (x, y) to (x, 4y), we can start by first graphing the given equation.

To graph, let's use sample points (x- and y-values):

x y

-2 2

-1 0.5

0 0

1 0.5

2 2

So we have the points (-2, 2), (-1, 0.5), (0, 0), (1, 0.5), and (2, 2) to help us graph the equation.

A transformation of (x, y) --> (x, ay) where a > 1 means a vertical stretch equal to |a|. In this case, because (x, y) is transformed to (x, 4y), the graph stretches vertically by a factor of 4.

To graph, let's use sample points (x- and y-values):

x y

-2 4(2) = 8

-1 4(0.5) = 2

0 4(0) = 0

1 4(0.5) = 2

2 4(2) = 8

The new graph would now look like this:

Given ABC shown below. Map ABC using the transformations given below. In each case, start with ABC , graph the image and state the Coodinates of the image's vertices.a) a reflection in the line x = 2 to produce A' B' C'b) a reflection in the line y= 1 to produce A" B"C"

Answers

a) A'(8,7), B' (10, -6) and C' (2,-3)

b) A" (-4,-5) B" (-6,8) C" (2,5)

1) Examining the graph, we can locate the following points of ABC

To reflect across line x=2 let's count to the left the same distance from x=2

Pre-image Reflection in the line x=2

A (-4, 7) (x+8, y) A'(8,7)

B (-6,-6) (x+16, y) B' (10, -6)

C (2,-3) (x,y) C' (2,-3) Remains the same since C is on x=2

b) A reflection about the line y=1 similarly we'll count the distances and then write new points over the line y=1.

So the Image of this is going to be

Pre-image Reflection in the line x=2

A (-4, 7) (x, y-12) A" (-4,-5)

B (-6,-6) (x, y) B" (-6, 8)

C (2,-3) (x,y) C' (2,5)

A" (-4,-5)

B" (-6,8)

C" (2,5)

3. A coin is tossed 140 times. The probability of getting tails is p = 0.500. Would a result of 55heads out of the 140 trials be considered usual or unusual? Why?Unusual, because the result is less than the maximum usual value.O Usual, because the result is between the minimum and maximum usual values.Unusual, because the result is less than the minimum usual value.Unusual, because the result is more than the maximum usual value

Answers

In order to calculate the minimum and maximum usual values, first let's calculate the mean and standard deviation of this distribution:

[tex]\begin{gathered} \mu=n\cdot p=140\cdot0.5=70\\ \\ \sigma=\sqrt{np(1-p)}=\sqrt{140\cdot0.5\cdot0.5}=5.92 \end{gathered}[/tex]

Now, calculating the minimum and maximum usual values, we have:

[tex]\begin{gathered} minimum=\mu-2\sigma=70-11.84=58.16\\ \\ maximum=\mu+2\sigma=70+11.84=81.84 \end{gathered}[/tex]

Since the given result is 55, it is an unusual reslt, because it is less tahan the minimum usual value.

Correct option: third one.

In the diagram below the larger angle is four times bigger than the smaller angle find the larger angle

Answers

Answer:

Given that,

In the diagram below the larger angle is four times bigger than the smaller angle

To find the larger angle.

Let x be smaller angle.

Then we get,

Larger angle is,

[tex]4x[/tex]

Larger angle and smaller angle are a linear pair.

Therefore we get,

[tex]x+4x=180[/tex][tex]5x=180[/tex][tex]x=36\degree[/tex]

Larger angle is,

[tex]4x=4\times36=144[/tex]

The larger angle is 144 degrees.

what is the y- intercept in the following equationy=-4x-5

Answers

[tex]\begin{gathered} y=-4x-5 \\ y=-4(0)-5 \\ y=0-5 \\ y=-5 \\ \text{the y-intercept is -5} \end{gathered}[/tex]

I need help quick with a math question !

Answers

Step-by-step explanation:

how do you solve for x in the following problem... 4 (x + 3) -2x + 8 = 28

Answers

Given the expression

[tex]4(x+3)-2x+8=28[/tex]

To solve it for x, the first step is to calculate the term in parentheses, for this you have to apply the distributive property of multiplication.

[tex]\begin{gathered} (4\cdot x)+(4\cdot3)-2x+8=28 \\ 4x+12-2x+8=28 \end{gathered}[/tex]

Next order the alike terms toghether and calculate:

[tex]\begin{gathered} 4x-2x+12+8=28 \\ 2x+20=28 \end{gathered}[/tex]

Subtract 20 to both sides of the equation:

[tex]\begin{gathered} 2x+20-20=28-20 \\ 2x=8 \end{gathered}[/tex]

And finally divide by 2 to reach the value of x:

[tex]\begin{gathered} \frac{2x}{2}=\frac{8}{2} \\ x=4 \end{gathered}[/tex]

For this equation x=4

If the number 659, 983 is rounded to the nearest hundred, how many zeros does the rounded number have?The solution is

Answers

We will have the following:

*For 569:

For this number we would round to 600, thus the number of zeros the rounded number would be 2.

*For 983:

For this number, we would round to 1000, thus the number of zeros the rounded number would be 3.

Cris pays a total of $11 for every 6 Gatoraid bottles. Circle the graph models a relationship with the same unit rate?

Answers

The line that describes this relationship goes from (0,0) to the point (6,11),we can draw it like this:

What is 6 hundred thousand in hundreds

Answers

600,000 (Six hundred thousand)

1) If we divide 600,000 by 100 we'll have 6000

So 600,000 is equal to 6000 hundreds.

Determine the equation of the line that passes through the point (-1, 2) and isperpendicular to the line y = -2.

Answers

[tex]y=-\frac{1}{2}x+\frac{3}{2}[/tex]

1) In this question, let's find the equation, using the point-slope formula:

[tex](y-y_0)=m(x-x_0)[/tex]

2) Notice that since we want a perpendicular line we can write a perpendicular line to y=2, as x=-1/2 for -1/2 is the opposite and reciprocal to 2 (the necessary condition to get a perpendicular line).

So, the slope of that perpendicular line is -1/2

3) Let's plug into that Point-Slope formula, the slope m= -1/2 and the point:

[tex]\begin{gathered} (y-2)=-\frac{1}{2}(x+1) \\ y-2=-\frac{1}{2}x-\frac{1}{2} \\ y=-\frac{1}{2}x-\frac{1}{2}+2 \\ y=-\frac{1}{2}x+\frac{3}{2} \end{gathered}[/tex]

4) Thus, the answer is:

[tex]y=-\frac{1}{2}x+\frac{3}{2}[/tex]

If ABCD is dilated by a factor of 1/2coordinate of d' would be

Answers

Given If the scale factor is 1/2, then every coordinate point of the original triangle is multiplied by the scale factor 1/2.[tex]\begin{gathered} D(2,-2) \\ Dialetion\text{ }\frac{1}{2}(2,-2) \\ \text{ } \\ \\ D^{\prime}(1,-1) \end{gathered}[/tex]The final answer[tex]D^{\prime}(1,-1)[/tex]

The area of a triangle is 2312 . Two of the side lengths are 93 and 96 and the included angle is obtuse. Find the measure of the included angle, to the nearest tenth of a degree.

Answers

to facilitate the exercise we will draw the triangle

We start using the area

[tex]A=\frac{b\times h}{2}[/tex]

where A is the area, b the base and h the height

if we replace A=2312 and b=96 we can calculate the height(h)

[tex]\begin{gathered} 2312=\frac{96\times h}{2} \\ \\ h=\frac{2312\times2}{96} \\ \\ h=\frac{289}{6} \end{gathered}[/tex]

now to calculate the measure of the angles we can solve the red triangle

first we find Y using trigonometric ratio of the sine

[tex]\sin (\alpha)=\frac{O}{H}[/tex]

where alpha is the reference angle, O the opposite side from the angle and H the hypotenuse of the triangle

using Y like reference angle and replacing

[tex]\sin (y)=\frac{\frac{289}{6}}{93}[/tex]

simplify

[tex]\sin (y)=\frac{289}{558}[/tex]

and solve for y

[tex]\begin{gathered} y=\sin ^{-1}(\frac{289}{558}) \\ \\ y=31.2 \end{gathered}[/tex]

value of angle y is 31.2°

Y and X are complementary because make a right line then if we add both numbers the solution is 180°

[tex]\begin{gathered} y+x=180 \\ 31.2+x=180 \end{gathered}[/tex]

and solve for x

[tex]\begin{gathered} x=180-31.2 \\ x=148.8 \end{gathered}[/tex]

measure of the included angle is 148.8°

(7x10^1(4x10^-7

(5.55 x 10^4) - ( 3.41 x 10^4)

(9 x 10^7) divided (3 x 10^3)

Work needs to be shows !!!

Answers

Answer:

(5.55 * 10^4) - (3.41 * 10^4)

=21,400

(9 * 10^7) divided (3 * 10^3)

= 30,000

Step-by-step explanation:

(5.55 * 10^4) - (3.41 * 10^4)

= (5.55 * 10,000) - (3.41 * 10,000)

= 55,500 - 34,100

= 21,400

(9 * 10^7) divided (3 * 10^3)

= (9 * 10,000,000) ÷ (3 * 1,000)

= 90,000,000 ÷ 3,000

= 30,000

Sorry but i don't understand the "(7x10^1(4x10^7". Your question is invalid.

Consider the following measures shown in the diagram with the circle centered at point A. Determine the arc length of CB.

Answers

Answer:

[tex]\frac{4}{3}\pi\; cm[/tex]

Explanation:

If an arc of a circle radius, r is subtended by a central angle, θ, then:

[tex]\text{Arc Length}=\frac{\theta}{360\degree}\times2\pi r[/tex]

In Circle A:

• The central angle, θ = 40 degrees

,

• Radius = 6cm

Therefore, the length of arc CB:

[tex]\begin{gathered} =\frac{40}{360}\times2\times6\times\pi \\ =\frac{4}{3}\pi\; cm \end{gathered}[/tex]

The correct choice is C.

Which expression is equivalent to sin(71(1) cos (72) - cos () sin (77.)?1?O cos (5)O sin (5)COS2012sin

Answers

[tex]\sin (\frac{\pi}{12})\cos (\frac{7\pi}{12})-\cos (\frac{\pi}{12})\sin (\frac{7\pi}{12})[/tex]

Let:

[tex]\begin{gathered} A=\frac{\pi}{12} \\ B=\frac{7\pi}{12} \end{gathered}[/tex]

Using the sine difference identity:

[tex]\begin{gathered} \sin (A)\cos (B)-\cos (A)\sin (B)=\sin (A-B) \\ so\colon \\ \sin (\frac{\pi}{12})\cos (\frac{7\pi}{12})-\cos (\frac{\pi}{12})\sin (\frac{7\pi}{12})=\sin (\frac{\pi}{12}-\frac{7\pi}{12}) \\ \sin (\frac{\pi}{12}-\frac{7\pi}{12})=\sin (-\frac{6\pi}{12}) \\ \sin (-\frac{\pi}{2}) \end{gathered}[/tex]

Answer:

[tex]\sin (-\frac{\pi}{2})[/tex]

a new car is purchased for 24800. the value of the car depreciates at 12% per year what is the Y intercept of starting value

Answers

EXPLANATION

Let's see the facts:

Purchase Price = $24,800

Depretiation = 12%/year

We should apply the formula for exponential decay wich is expressed as:

A = P(1-r)^t

A= value of the car after t years

t= number of years

P = Initial Value

r= rate of decay in decimal form

We have that A=unknown t=? P=24,800 r=0.12

Replacing terms:

A=24,800(1-0.12)^t

Now, the y-intercept is the value obtained when t=0, so substituting this on the equation give us the following result:

A=24,800(1-0.12)^0 = 24,800(0.88)^0= 24,800*1=24,800

The starting value is $24,800

A paper is sold for Php60.00, which is 150% of the cost. How much is the store's cost?

Answers

The store's cost is php40

Let's call the store Cost = C

This means that this cost is elevated a 150% in order to get the price of php60

In an mathematical expression, this is:

C · 150% = php60

Then, let's convert the percentage to decimal. To do this, we just divide the percentage by 100:

150% ÷ 100 = 1.5

Now we can solve:

[tex]\begin{gathered} C\cdot1.5=60 \\ C=\frac{60}{1.5}=40 \end{gathered}[/tex]

Then the store cost is C = php40

Given:• AJKL is an equilateral triangle.• N is the midpoint of JK.• JL 24.What is the length of NL?L24JKNO 12O 8V3O 12V2O 1213

Answers

Answer:

12√3

Explanation:

First, we know that JL = 24.

Then, the triangle JKL is equilateral. It means that all the sides are equal, so JK is also equal to 24.

Finally, N is the midpoint of segment JK, so it divides the segment JK into two equal parts. Therefore, JN = 12.

Now, we have a right triangle JLN, where JL = 24 and JN = 12.

Then, we can use the Pythagorean theorem to find the third side of the triangle, so NL is equal to:

[tex]\begin{gathered} NL=\sqrt[]{(JL)^2-(JN)^2} \\ NL=\sqrt[]{24^2-12^2} \end{gathered}[/tex]

Because JL is the hypotenuse of the triangle and JN and NL are the legs.

So, solving for NL, we get:

[tex]\begin{gathered} NL=\sqrt[]{576-144} \\ NL=\sqrt[]{432} \\ NL=\sqrt[]{144(3)} \\ NL=\sqrt[]{144}\cdot\sqrt[]{3} \\ NL=12\sqrt[]{3} \end{gathered}[/tex]

Therefore, the length of NL is 12√3

11) -3(1 + 6r) = 14 - r

Answers

[tex]-3(1+6r)=14-r[/tex]

Distributing over parentheses,

[tex]\begin{gathered} -3\cdot1+(-3)\cdot6r=14-r \\ -3-18r=14-r \end{gathered}[/tex]

Adding r at both sides,

[tex]\begin{gathered} -3-18r+r=14-r+r \\ -3-17r=14 \end{gathered}[/tex]

Adding 3 at both sides,

[tex]\begin{gathered} -3-17r+3=14+3 \\ -17r=17 \end{gathered}[/tex]

Dividing by -17 at both sides,

[tex]\begin{gathered} \frac{-17r}{-17}=\frac{17}{-17} \\ r=-1 \end{gathered}[/tex]

Match each solid cone to it’s surface area. Answers are rounded to the nearest square unit

Answers

The surface area of a cone is given by the formula below:

[tex]S=\pi r^2+\pi rs[/tex]

Where r is the base radius and s is the slant height.

So, calculating the surface area of first cone, we have:

[tex]\begin{gathered} s^2=21^2+6^2\\ \\ s^2=441+36\\ \\ s^2=477\\ \\ s=21.84\\ \\ S=\pi\cdot6^2+\pi\cdot6\cdot21.84\\ \\ S=525 \end{gathered}[/tex]

The surface area of the second cone is:

[tex]\begin{gathered} s^2=8^2+12^2\\ \\ s^2=64+144\\ \\ s^2=208\\ \\ s=14.42\\ \\ S=\pi\cdot12^2+\pi\cdot12\cdot14.42\\ \\ S=996 \end{gathered}[/tex]

The surface area of the third cone is:

[tex]\begin{gathered} s^2=15^2+8^2\\ \\ s^2=225+64\\ \\ s^2=289\\ \\ s=17\\ \\ S=\pi\cdot8^2+\pi\cdot8\cdot17\\ \\ S=628 \end{gathered}[/tex]

And the surface area of the fourth cone is:

[tex]\begin{gathered} s^2=10^2+10^2\\ \\ s^2=100+100\\ \\ s^2=200\\ \\ s=14.14\\ \\ S=\pi\cdot10^2+\pi\cdot10\cdot14.14\\ \\ S=758 \end{gathered}[/tex]

Yolanda has a rectangular poster that is 16 cm long and 10 cm wide what is the area of the poster in square meters do not round your answer is sure to include the correct unit in your answer

Answers

The area of a rectangle can be calculated as the height times the wide.

But be careful, the problem asks it in square meters! So let's use meters instead of centimeters.

Remember that : 1 m = 100 cm ----> 1 cm = 0.01 m

[tex]\begin{gathered} A=b\cdot h \\ \\ A=0.16\cdot0.10 \end{gathered}[/tex]

Doing the multiplication

[tex]A=0.016\text{ m}^2[/tex]

Therefore the area of the poster is 0.016 square meters

If the figure below were reflected across the waxis, what would be the new coordinates of point A

Answers

The coordinates of point A are (-2,3). A reflection across the y-axis is given by:

[tex](x,y)\rightarrow(-x,y)[/tex]

Applying this rule to point A we have:

[tex](-2,3)\rightarrow(2,3)[/tex]

Therefore, the image of point A is (2,3) and the correct option is B.

Find the missing factor. 8x2 - X - 9 = (x + 1)(

Answers

In this case, we'll have to carry out several steps to find the solution.

Step 01:

Data

8x² - x - 9 = (x + 1) ( ? )

Step 02:

We must find the missing root to solve the exercise.

x1 = - 1

x2 :

a = 8

b = -1

c = -9

[tex]x\text{ = }\frac{-(-1)\pm\sqrt[]{(-1)^2-4\cdot8\cdot(-9)}_{}}{2\cdot8}[/tex][tex]x\text{ = }\frac{1\pm17}{16}[/tex][tex]x\text{ = }\frac{1+17}{16}=\frac{18}{16}=\frac{9}{8}[/tex]

x2 = 9 / 8

The answer is:

( x - 9/8)

8x² - x - 9 = (x + 1) ( x - 9/8 )

In a mid-size company, the distribution of the number of phone calls answered each day by each of the 12 receptionists is bell-shaped and has a mean of 44 and a standard deviation of 4. Using the empirical rule, what is the approximate percentage of daily phone calls numbering between 36 and 52?

Answers

The empirical rule is an approximation that can be used sometimes if we have data in a normal distribution. If we know the mean and standard deviation, we can use the rule to approximate the percentage of the data that is 1, 2, and 3 standard deviations from the mean. The rules is:

In this case, the mean is 44. The receptionist who answered less than 44 phone calls are to the left of the mean, and to the right are the ones who answered more. Since we want to know the percentage of phone calls numbering between 36 and 52, we know that:

[tex]\begin{gathered} 44+4=48 \\ . \\ 48+4=52 \end{gathered}[/tex][tex]\begin{gathered} 44-4=40 \\ . \\ 40-4=36 \end{gathered}[/tex]

Thus, the lower bound is two standard deviations from the mean, and the upper bond is also 2 standard deviations from the mean.

Using the chart above, we can see that this corresponds to approximately 95% of the data.

The answer is approximately 95% of the data is numbering between 36 and 52

During a coffee house's grand opening.350 out of the first 500 customers who visited ordered only one single item while the rest ordered multiple items. Among the 150 customs who left a tip 60 of them ordered multiple items?

Answers

We are given a two-way frequency table with some missing joint frequencies.

The frequencies in the total row and column are called "marginal frequencies"

The frequencies in the other rows and columns are called "joint frequencies"

Let us first find the joint frequency "Single Item and Tip"

[tex]\begin{gathered} x+60=150 \\ x=150-60 \\ x=90 \end{gathered}[/tex]

So, the joint frequency "Single Item and Tip" is 90 (option B)

Now, let us find the joint frequency "Single Item and No Tip"

[tex]\begin{gathered} 90+x=360 \\ x=360-90 \\ x=270 \end{gathered}[/tex]

So, the joint frequency "Single Item and No Tip" is 270 (option D)

Now first we need to find the marginal frequency as below

[tex]\begin{gathered} 360+x=500 \\ x=500-360 \\ x=140 \end{gathered}[/tex]

Finally, now we can find the joint frequency "Multiple Items and No Tip"

[tex]\begin{gathered} 60+x=140 \\ x=140-60 \\ x=80 \end{gathered}[/tex]

So, the joint frequency "Multiple Items and No Tip" is 80 (option A)

Therefore, the missing joint frequencies are

Option A

Option B

Option D

I need help solving this practice problem If you can , answer (a) and (b) separately so I can tell which is which

Answers

Step 1:

Write the expression

[tex](3x^5\text{ - }\frac{1}{9}y^3)^4[/tex]

Step 2:

a)

[tex]\begin{gathered} (3x^5\text{ - }\frac{1}{9}y^3)^4 \\ =^4C_0(3x^5)^4(-\frac{1}{9}y^3)^0+^4C_1(3x^5)^3(-\frac{1}{9}y^3)^1+^4C_2(3x^5)^2(-\frac{1}{9}y^3)^2+ \\ +^4C_1(3x^5)^1(-\frac{1}{9}y^3)^3+^4C_0(3x^5_{})^0(-\frac{1}{9}y^3)^4 \end{gathered}[/tex]

Step 3:

b) simplified terms of the expression

[tex]\begin{gathered} Note\colon \\ ^4C_0\text{ = 1} \\ ^4C_1\text{ = 4} \\ ^4C_2\text{ = 6} \\ ^4C_3\text{ = 4} \\ ^4C_4\text{ = 1} \end{gathered}[/tex]

Next, substitute in the expression

[tex]\begin{gathered} =\text{ 1}\times81x^{20}\times1\text{ - 4}\times27x^{15}\text{ }\times\text{ }\frac{y^3}{9}\text{ + 6 }\times9x^{10}\times\frac{y^6}{81}\text{ - 4}\times3x^5\text{ }\times\text{ }\frac{y^9}{729} \\ +\text{ 1 }\times\text{ 1 }\times\frac{y^{12}}{6561}\text{ } \end{gathered}[/tex][tex]=81x^{20}-12x^{15}y^3\text{ + }\frac{2}{3}x^{10}y^6\text{ - }\frac{4}{243}x^5y^9\text{ + }\frac{1}{6561}y^{12}[/tex]


One year there was a total of 44 commercial and noncommercial orbital launches worldwide. In addition, the number number of commercial orbital launches. Determine the number of commercial and noncommercial orbital launches was two more thank twice the number of commercial orbital launches (HURRY I NEED ANSWER)

Answers

The number of commercial orbital is 14, and the number of noncommercial orbital is 30.

What is algebra?

When numbers and quantities are represented in formulas and equations by letters and other universal symbols.

Given that,

The total number of commercial and noncommercial orbital launches worldwide = 44

Also, the number of noncommercial orbital is two more than twice of commercial orbital

Let the number of commercial orbital =x

Then number of noncommercial orbital = 2x+2

Since, total number of commercial and noncommercial orbital = 44

x + 2x +2 = 44

3x = 42

x = 14

The number of commercial orbital = x = 14

The number of noncommercial orbital = 2x+2 = 2×14+2 = 30

To know more about Algebra on:

https://brainly.com/question/24875240

#SPJ1

Other Questions
What is the solution to the equation k - 4 3/4 = 8 1/4?k = 4 1\2k = 12k = 13k = 4 How did citizens united change campaign finance laws? select three correct answers. Write an equation of the line that passes through a pair of points: 5 4 37 2 1+ 4 -3 -2 -1 1 -3 a. y = x + 3 b. y = x - 3 C. y = -x + 2 d. y = -x-2 Please select the best answer from the choicon A scientist observes and counts 155 bacteria in a culture. Later, the scientist counts again and finds the number has increased as shown. a 150. g baseball is thrown with a speed of 20. m/s. it is hit straight back toward the pitcher at a speed of 40 m/s. if the baseball and bat are in contact for 6.0 ms, what is the magnitude of the average force delivered to the ball? find the image of (-2,-9) after a reflection over the y-axis The diagram shows how 6-foot boards and 8-foot boards are joined to form rectangular frames in a wall. Which is closest to the length of the diagonal brace for the wall? 6 ft 8 ft A. 10 ft B. 12 ft C. 13 ft D. 11 ft (Right angle) Trigonometry Help me find the X value please! Which is NOT true?a) 9+4=17-4b)8+7=14+3c)11=19-8d)5+8=20-7 if you are paid $4.50 per hour, how many hours will you have to work to earn $1000.00 A company purchased 10,000 pairs of men's slacks for $19.16 per pair and marked them up $22.43. What was the selling price of each pair of slacks? Use the formula S=C+M dividing 5 by 10 + 1 If 15.2g of aluminum reacts with 39.1g of chlorine, how many g of AlCl3 forms? 17. Show your work-Factor the expression: 35x+63 * Your answer What is the total surface area ratio of following similar solids?30 mi45 mi60 mi90 mi09:4O 15:6O 5:4O 3:2 need some help pls due date 10pmthanks to whoever answers what is the answer1-m=6-6m Solve the equation for x, and enter your answer below.3x-3 + 5x = 37 A python (p) is 3.9 feet longer than a boa constrictor (6).Select an expression from each box to create an equation that compares the lengths of the snakes Consider a gas at (3.420x10^2) kPa in a (7.6100x10^-1) L container at (1.8000x10^1)C. What will thpressure of the gas be. if it is released into a room with a volume of (1.91x10^4) L and a temperature(1.3000x10^1)C? Express your answer to three significant digits.